Aplicando el teorema de los residuos a una función de correlación donde la función de Fermi no tiene polos

Dejar norte F ( ω ) = 1 mi β ( ω ) + 1 Sea la función de Fermi.

Una función de correlación de depósito fermiónico viene dada por:

(5) C 12 ( t ) = + d ω   j R ( ω ) norte F ( ω ) mi i ω t

La función de Fermi aquí se da en términos de polinomios de Chebyshev.

Los coeficientes de los polinomios de Chebyshev vienen dados por:

C k = 2 π 0 π F ( porque θ ) porque ( k θ ) d θ
Y la función de Fermi en sí misma es (Chebyshev se aproximó, por así decirlo):

(6) norte F ( X ) = k = 0 norte [ 2 π 0 π porque k θ d θ mi β ( mi F porque θ ) + 1 ] T k ( X )
Dónde T k ( X ) son los polinomios de Chebyshev del primer tipo.

Ahora, dado que la función de Fermi aquí no tiene polos (ya que se da en términos de polinomios de interpolación de Chebyshev que no tienen polos), los polos en la función de correlación son solo los de la densidad espectral:

(7) j R ( ω ) = k = 1 metro pag k 4 Ω k ( ω Ω k ) 2 + Γ k 2

Solo hay poste en: ω = Ω k i Γ k = Ω k , y el residuo es 1 ( ω Ω k ) i Γ k | ω = Ω k = 1 2 i Γ k .

O el residuo de j R ( ω ) en ω = Ω k :

(8) res ω = Ω k j R ( ω ) = pag k 4 Ω k ( 2 i Γ k )

Mi pregunta es: ¿Cómo se puede resolver ahora la integral de la función de correlación usando el teorema de los residuos/lema de Jordan? ¿Todavía es posible o se debe emplear otro esquema?

Si la función de Fermi se diera en términos de la suma de frecuencias de Matsubara, habría tenido polos y luego podrían calcularse sus residuos. Ahora no es así, y no puedo ver cómo se puede resolver la integral ahora. Si la función de Fermi tuviera polos, podríamos haber dicho:

Tomando nota de que los polacos de j R ( ω ) : Ω k , y los polos de norte F ( ω ) : v k , podríamos haber obtenido:

(9) C 12 ( t ) = ( ) ( 2 i π ) { k = 1 metro res ω = Ω k [ j R ( ω ) ] norte F ( Ω k ) mi i Ω k t + k res ω = v k [ norte F ( ω ) ] j R ( v k ) mi i v k t }

Y entonces se podrían haber calculado los residuos.

Una versión un poco diferente del mismo problema (si decide responder, responda esto primero):

Aquí hay una función:

(1) C 12 ( t ) = + d ω   j R ( ω ) norte F ( ω ) mi i ω t

Aquí está norte F ( X ) norte F ( ω ) :

(2) norte F ( X ) = k = 0 norte [ 2 π 0 π porque k θ d θ mi β ( mi F porque θ ) + 1 ] T k ( X )
Dónde T k ( X ) son los polinomios de Chebyshev del primer tipo.

También, norte F ( X ) no tiene polos.

Y,

(3) j R ( ω ) = k = 1 metro pag k 4 Ω k ( ω Ω k ) 2 + Γ k 2
Dónde Ω , P y Γ son solo algunos números.

¿Se cumplen los prerrequisitos del lema de Jordan? Es decir, ¿puede la ecuación (1) escribirse como la ecuación (4) después de insertar (2) y (3) en (1) y luego aplicar el Lema de Jordan?

(4) C 12 ( t ) = ( ) ( 2 i π ) { k = 1 metro res ω = Ω k [ j R ( ω ) ] norte F ( Ω k ) mi i Ω k t }

Probablemente me estoy perdiendo algo, pero ¿por qué es un problema tener menos polos? Todavía cierra el contorno con un ciclo de contribución 0 y luego encuentra los residuos del integrando, sean lo que sean, dentro del contorno cerrado. Si no quedan residuos de norte F para resumir, entonces que así sea: todavía hay otros residuos para el integrando como un todo, y no es como norte F no tuvo efecto en la respuesta.
Eso parece algo que podría ser cierto. Pero la cuestión es que ¿podemos seguir aplicando el teorema de los residuos/lema de Jordan si no hay polos? Si es posible, como dices, entonces solo tengo que evaluar norte F sobre el polo de la densidad espectral y listo, ¿verdad?
Sabes qué, tienes razón. Es posible cerrar el contorno y evaluar los residuos de j X ( ω ) solo. Pero que le pasa norte F entonces... justo lo que escribí arriba o algo más.
Pregunta relacionada por OP: physics.stackexchange.com/q/79521/2451
No veo ω en la derecha de su expansión de la función de Fermi (la ecuación no numerada después de la ecuación 2)
Sí, porque ha sido reemplazado por C o s θ . consulte esto: math.stackexchange.com/questions/532103/…
@Hasan, indique la sustitución aquí.
Todavía hay algo mal con la expresión después de la ecuación. 2 -- θ está integrado, no debe aparecer en la izquierda X se usa en rhs pero no está definido.
Está bien, voy a tener que pedirte que me perdones por eso. Solo llamemos norte F ( C o s θ + mi F ) una aproximación de la función fermi, tal que solo depende de x. Que en realidad es. Voy a cambiar.

Respuestas (3)

Acabo de descubrir que es fundamentalmente incorrecto aproximar la función de Fermi con polinomios de Chebyshev. La representación de la función de Fermi en el eje real mediante polinomios de Chebychev podría estar bien, pero la representación de la función de Fermi en el plano complejo y especialmente cerca de los polos de la función de Fermi es ciertamente muy pobre (no polo contra polo). Al aplicar el lema de Jordan a esta mala representación, nos perdemos los ingredientes más importantes, es decir, los polos, sin duda darán como resultado un resultado pobre.

¿Adónde quieres llegar con la aproximación de Chebyshev? Cada T k satisface

( 1 X 2 ) T k ( X ) X T k ( X ) + k 2 T k ( X ) = 0

así que desde ( 2 )

(*) [ ( 1 X 2 ) ( d d X ) 2 X d d X ] norte F ( X ) = k = 0 norte { 2 π 0 π porque ( k θ ) d θ mi β ( porque θ + mi F ) + 1 [ ( 1 X 2 ) ( d d X ) 2 X d d X ] T k ( X ) } = k = 0 norte { 2 π 0 π porque ( k θ ) d θ mi β ( porque θ + mi F ) + 1 [ k 2 T k ( X ) ] }

Para

C k = 2 π 0 π porque ( k θ ) norte F ( porque θ + mi F ) d θ

obtenemos por integración parcial doble

( mi F ) 2 C k = 2 π mi F 0 π porque ( k θ ) norte ˙ F ( porque θ + mi F ) d θ = 2 π mi F { 0 π k pecado ( k θ ) norte F ( porque θ + mi F ) d θ + [ porque ( k θ ) norte F ( porque θ + mi F ) ] 0 π } = 2 π { 0 π k pecado ( k θ ) norte ˙ F ( porque θ + mi F ) d θ + mi F [ ( 1 ) k norte F ( 1 + mi F ) norte F ( 1 + mi F ) ] } = 2 π { 0 π k 2 porque ( k θ ) norte F ( porque θ + mi F ) d θ + [ k pecado ( k θ ) norte F ( porque θ + mi F ) ] 0 π + [ ( 1 ) k norte ˙ F ( 1 + mi F ) norte ˙ F ( 1 + mi F ) ] } = k 2 C k + 2 π [ norte ˙ F ( 1 + mi F ) + ( 1 ) k norte ˙ F ( 1 + mi F ) ] = k 2 C k 2 β ˙ π [ mi β ( 1 + mi F ) norte F 2 ( 1 + mi F ) ( 1 ) k mi β ( 1 + mi F ) norte F 2 ( 1 + mi F ) ]

(el punto denota la derivada habitual). Juntos con ( ) obtenemos

[ ( 1 X 2 ) ( d d X ) 2 X d d X ] norte F ( X ) = k = 0 norte { ( mi F ) 2 C k + 2 β ˙ π [ ] } T k ( X ) = ( mi F ) 2 norte F ( X ) + 2 β ˙ π [ mi β ( 1 + mi F ) norte F 2 ( 1 + mi F ) k = 0 norte T k ( X ) k = 0 norte mi β ( 1 + mi F ) norte F 2 ( 1 + mi F ) ( 1 ) k T k ( X ) ]

Si estima que el plazo 'largo' es insignificante, entonces

[ ( 1 X 2 ) ( d d X ) 2 X d d X ] norte F ( X ) ( mi F ) 2 norte F ( X )

No sé si esto ayuda.

Lo que hiciste puede ser correcto, pero es irrelevante.

Necesitas un teorema de la forma:

  • dado H(z)=F(z)G(z),
  • F(z) es analítica en todas partes (esta es la distribución de Fermi),
  • G(z) tiene un polo en z 0 (esta es su función de correlación)

Entonces

  • R mi s [ H ( z 0 ) ] = F ( z 0 ) R mi s [ GRAMO ( z 0 ) ]

Esto se sigue de la definición del residuo como

R mi s [ H ( z 0 ) ] = límite z z 0 ( z z 0 ) H ( z ) = [ yo i metro z z 0 F ( z ) ] [ yo i metro z z 0 ( z z 0 ) GRAMO ( z ) ] = F ( z 0 ) R mi s [ GRAMO ( z 0 ) ]

donde puedo sacar el F fuera del límite ya que sé que es analítico en todas partes.

Pero la función de correlación y la función de Fermi no se multiplican así. La función de correlación contiene una integral de la función de Fermi multiplicada por la densidad espectral. ecuación (1).
Pensé que tu objetivo era evaluar la integral en (1) a través del teorema de los residuos; para mí parece que el integrando es j × norte × mi i ω t
Sí, eso es lo que es... pero J es la densidad espectral y tiene un polo. Entonces, lo que hice fue, (si miras (4)), tomar solo la primera suma en (4) y evaluar la función Fermi aproximada de Chebyshev sobre el polo de J. ¿Es esto lo que quieres decir también?
Lo que digo es que sabemos que la función de Fermi no tiene polos ; por lo tanto, no hay términos en la segunda suma de Eq. (3).
Sí, eso es verdad.
Entonces, ¿qué pasa con lo que dije en el comentario anterior? Eso es lo que sucederá si tomamos la primera suma en (4). ¿Es eso correcto? Eso es lo que escribiste en tu respuesta también: ( F ( z 0 ) ). ¿Estás seguro de que esto es correcto? (perdón por preguntar de esta manera) pero hay ciertas cosas que no puedo explicar (/entender correctamente) que hacen que esto parezca extraño.